Can the Rationals be Contained in Open Intervals with Infinitely Small Width?

Click For Summary
The discussion centers on proving that all rational numbers can be contained within open intervals whose total width is less than any given ε > 0. Participants note that since the rationals are countable, it's possible to create intervals around each rational number such that their combined width meets the ε constraint. Suggestions for constructing a converging series to achieve this include using terms like ε/(n^2+n) or ε*(1/2)^n. The focus is on demonstrating the convergence of the series to ε, which is crucial for the proof. Overall, the conversation emphasizes the relationship between countability and the ability to fit rationals within arbitrarily small widths.
zhang128
Messages
4
Reaction score
0

Homework Statement


Prove that the rationals as a subset of the reals can all be contained in open intervals the sum of whose width is less than any \epsilon > 0.


Homework Equations





The Attempt at a Solution

 
Physics news on Phys.org
You aren't playing the game here. What do you think about the problem? You can't leave the Attempt at a Solution completely blank.
 
ups, my bad. I was thinking that since rationals are countable, I just need to make the open interval around each rational such that the total sum is less than \epsilon. Then the question turns to prove the sum of a finite series converges to the epsilon??
 
zhang128 said:
ups, my bad. I was thinking that since rationals are countable, I just need to make the open interval around each rational such that the total sum is less than \epsilon. Then the question turns to prove the sum of a finite series converges to the epsilon??

Much better, thanks. Can you write a series that converges to epsilon? If you can write a series that sums to say, 1, you should be able to write a series that converges to epsilon.
 
hmmm, like epsilon/(n^2+n)??
 
zhang128 said:
hmmm, like epsilon/(n^2+n)??

Sure, that works. I would have said sum epsilon*(1/2)^n. But whatever you like.
 
Question: A clock's minute hand has length 4 and its hour hand has length 3. What is the distance between the tips at the moment when it is increasing most rapidly?(Putnam Exam Question) Answer: Making assumption that both the hands moves at constant angular velocities, the answer is ## \sqrt{7} .## But don't you think this assumption is somewhat doubtful and wrong?

Similar threads

  • · Replies 9 ·
Replies
9
Views
2K
  • · Replies 18 ·
Replies
18
Views
2K
Replies
10
Views
2K
Replies
7
Views
2K
Replies
1
Views
2K
  • · Replies 2 ·
Replies
2
Views
1K
  • · Replies 5 ·
Replies
5
Views
2K
  • · Replies 4 ·
Replies
4
Views
2K
  • · Replies 25 ·
Replies
25
Views
3K
Replies
2
Views
2K